PLEASE HELP ILL GIVE EXTRA POINTS FOR ANSWER ⭐️⭐️⭐️‼️‼️

PLEASE HELP ILL GIVE EXTRA POINTS FOR ANSWER

Answers

Answer 1

Answer:

d

Step-by-step explanation:

area=length ×width

156=13×w

w=156/13=?

P=2(l+w)

P=2(13+w)


Related Questions

Four-sevenths of the people in a room are seated in seven-eighths of the chairs. The rest of the people are standing. If there are 8 empty chairs, how many people are in the room?

Answers

Answer:

147

Step-by-step explanation:

4/7 people in chairs

7/8 of chairs, rest are standing

8 empty chairs

empty chairs = 1/8

full chairs = 8 x 8

7/8 = 64.

4/7 = 64

1/7 = 64 / 4

1/7 = 21

7/7 = 21 x 7

7/7 = 147

There are 147 people in the room.

There are 147 people in the room.

The given is,

Four-sevenths of the people in a room are seated in seven-eighths of the chairs.

The rest of the people are standing.

there are 8 empty chairs, how many people are in the room in a room are seated in seven-eighths of the chairs.

The rest of the people are standing.

We have to determine there are 8 empty chairs, how many people are in the room.

Four-sevenths of the people.

What is the standard form of the Four-sevenths?

4/7 people in chairs.

7/8 of the chairs, rest are standing

The 8 empty chairs,

The empty chairs = 1/8

The full chairs = 8 x 8

7/8 = 64.

4/7 = 64

1/7 = 64 / 4

1/7 = 21

7/7 = 21 x 7

7/7 = 147

There are 147 people in the room.

To learn more about the unknown quantity visit:

https://brainly.com/question/12247728

A certain brand of laundry detergent is manufactured for $3.85 and then marked up 40% by the local store before selling. The store offers a 20% discount with the purchase of three or more bottles of the detergent. Showing all work, determine the total cost, before tax, of five bottles of detergent. (10 points)

Answers

Answer:

$21.56

Step-by-step explanation:

Original price = $3.85Marked up by 40% = 3.85 x 1.4 = $5.39 per bottle

Cost and discount of 5 bottles

Cost = 5 x 5.39 = $26.95Discount of 20% = 26.95 x 0.8 = $21.56

Answer:

$21.56

Step-by-step explanation:

Original cost of one bottle of detergent = $3.85

Cost of one bottle after 40% mark up = 1.4 × $3.85 = $5.39

Cost of 5 bottles = 5 × $5.39 = $26.95

Cost of 5 bottles after applying 20% discount = 0.8 × $26.95 = $21.56

Therefore, the total cost of 5 bottles of detergent is $21.56

------------------------------------------------------------------------------------------------

Information about percetange increase/decrease

If you increase the price of something by 40% then it will be 140% of the original price (the original price is 100% and 100% + 40% = 140%).

Convert to decimal form:  140% = 140/100 = 1.4

Therefore, to calculate an increase of 40%, multiply the original price by 1.4

If you decrease the price of something by 20% then it will be 80% of the previous price (the original price is 100% and 100% - 20% = 80%).

Convert to decimal form:  80% = 80/100 = 0.8

Therefore, to calculate a decrease of 20%, multiply the previous price by 0.8

Solve this system of equations
3x+3y+z =-18
x-3y+2z=15
8x-2y+3z=7

Answers

just showed this but here again

A box has length 3 ft, width 5 ft, and height 2 ft.

What is the volume?
cu ft

Answers

Answer:

30 cubic feet

Step-by-step explanation:

The volume of a box is found by multiplying the area of the base by the height.  The area of the base is found by multiplying the length by the width.  (3 x 5)(2) = 15(2) = 30 cubic feet

I am so confused... I need help

Answers

Ima say C

Cause there is two bases, top and bottom, and then the two sides

If you drew a marble from a bag containing 40 marbles and you had a probability of drawing a red marble, how many red marbles are in the bag?

6
10
3
5

Answers

Answer:

There are 5 red Marbles

Step-by-step explanation:

Alright so since the probability of drawing a red marble is

P(Red) = #Outcomes of Red/#Outcomes in Sample space

The sample Space is 40 marbles.

The probability of drawing red was said to be [tex]\frac{1}{8}[/tex]

P(Red) = [tex]\frac{1}{8}[/tex]

[tex]P(Red) = \frac{r}{40}\\ \frac{1}{8} = \frac{r}{40}\\ \frac{40}{8} = r\\ 5 = r[/tex]

Mr. and Mrs. Williams hope to send their daughter to college in twelve years. How much money should they invest now at an interest rate of 8.5% per year,
compounded continuously, in order to be able to contribute $9000 to her education?

Answers

[tex]~~~~~~ \textit{Continuously Compounding Interest Earned Amount} \\\\ A=Pe^{rt}\qquad \begin{cases} A=\textit{accumulated amount}\dotfill & \$9000\\ P=\textit{original amount deposited}\\ r=rate\to 8.5\%\to \frac{8.5}{100}\dotfill &0.085\\ t=years\dotfill &12 \end{cases} \\\\\\ 9000=Pe^{0.085\cdot 12}\implies 9000Pe^{1.02}\implies \cfrac{9000}{e^{1.02}}=P\implies 3245.35\approx P[/tex]

Find the value of x. Enter only the number.

Answers

Answer: 5

Step-by-step explanation: These two triangles are similar triangles. Since they are similar, they must be scaled equivalently. The scale factor is 5. To know this, 3 times 5 is 15, and being equivalent, we can figure 5 times 5 is 25. Therefore x is 5. That also means that the scale factor of the smallest triangle to the biggest triangle is 5. Thus, x is 5.

Please help
Simplify:

Answers

Answer:

-9 3/4a+10 2/5

Step-by-step explanation:

-12 * 5/6a = -10a

-12 * -7/8 =  10 1/2

1/14 * 3 1/2a = 1/4 a

1/14 * -1 2/5 = -1/10

Put it together: -10a + 10 1/2 + 1/4a - 1/10

-10a+1/4a = -9 3/4a

10 1/2-1/10= 10 2/5

-9 3/4a + 10 2/5

It takes Marty 1 1/4 hours to get ready for school. If 1/5 of that time is used to shower and 1/3 of that time is used to eat breakfast, how long does it take him to shower and eat breakfast?

Answers

Answer: 40 minutes

Step-by-step explanation:

First, break this down into minutes. 1 1/4 hours is 75 minutes. 75 divided by 5 is 15. he spends 15 minutes taking a shower. 75 divided by 3 is 25. he spends 25 minutes eating breakfast. 25 plus 15 is 40. he spends 40 minutes to shower and eat breakfast.

This is a img thank you

Answers

Answer: A = and then below is incorrect then state how to find the correct answer and why it is correct.

Step-by-step explanation: hope it helps

Seventh grade

A triangular prism is 40 inches long and has a triangular face with a base of 42 inches and a
height of 28 inches. The other two sides of the triangle are each 35 inches. What is the
surface area of the triangular prism?

Answers

Step-by-step explanation:

area of 2 triangles + 3 rectangles

2 (0.5×42×28)

=2×588

=1176 inches square

2(35x40)=2800

42×40= + 1680

--------

=4480 inches square

1176+4480

=5656 square units

Find the x- and y-intercepts of y=−x/2−1.

Answers

Step-by-step explanation:

y = -x/2 - 1

the y-intercept is right there : -1.

it is the y-value when x = 0

the point of y-intercept is therefore (0, -1).

the x-intercept is the other way around. it is the x- value when y = 0.

0 = -x/2 - 1

1 = -x/2

-2 = x

the point of x-intercept is therefore (-2, 0)

Which describes the combined variation in the formula h=v/ πr^2?

h varies directly with V and inversely with the square of r.

h varies directly with V and inversely with π and the square of r.

h varies jointly with V and π, and inversely with the square of r.

h varies jointly with V and r2.

Answers

Step-by-step explanation:

h varies directly with v and inversely with the square of r.

1/pi is the constant of variation.

An exponential function has a growth factor or 3.76. What is the percentage growth rate?

Answers

The exponential function with a growth factor has a rate greater than 1

The percentage growth rate is 276%

How to determine the percentage growth rate?

The growth factor (b) is given as:

b = 3.76

The growth factor (b) is greater than 1.

So, the percentage growth rate (r) is calculated as:

r = b - 1

Substitute known values

r = 3.76 - 1

Evaluate the difference

r = 2.76

Express as percentage

r = 276%

Hence, the percentage growth rate is 276%

Read more about exponential functions at:

https://brainly.com/question/11464095

What is the area of the parallelogram shown? A) 60in B) 120in C) 140in D) 180in. + What is the area of a parallelogram that has a base 5 inches longer and the height 5 inches taller than a parallelogram shown? A) 230in B) 260in C) 290in D) 310in.

Answers

120 in sq.  ,   260 in sq.Step-by-step explanation:question 1.area of a parralelogram = perperndicular height x basetherefore area = 8in x 15 inarea is 120in sq.question 2.the second parallelogram dimensions are as followsheight = 8 in +5 in = 13 inbase = 15in + 5in = 20therefore area = 13x20area = 260 in sq.

I NEED HELP WITH MY HOMEWORK​​

Answers

Answer:

I-9I

Step-by-step explanation:

Absolute value (the straight brackets) is how far away the number is from zero, this makes -9 into positive 9

Answer:

C. |-9|

Step-by-step explanation:

|-9| = 9

9 > 7 > |5| (5) > -12

What is 88 3/4 rounded to the nearest whole number

Answers

Answer:

89

Step-by-step explanation:

df fd

Which expression is equivalent to 4.x? - 16?
O A. x² – 12
.
ОВ.
(2x + 4)(2x - 4)
Ос.
2x + 2x – 8 - 8
O D.
(2x - 4)(2x - 4)

Answers

Answer:    A

Step-by-step explanation:

Two parents were shopping online for
new clothes for their kids. One mom
spent $26.50 on 3 shirts and 1 pair of
pants. Another mom spent $42 on 4 of
the same shirts and 2 of the same pairs
of the same pants. What was the cost
of each shirt and pair of pants?

Answers

Answer:

Shirt= $5.50

Pant=$10

Step-by-step explanation:

Let's shirt be 'x' and pant be 'y'

=}. 3x + y = $26.50

4x + 2y = $42

=}. 3x + y = $26.50

2x + y = $21. ( then first term minus

second term )

=}. x = $ 5.50

Now, y = $26.50 - 3x

= $26.50 - 16.50

= $10

Thx I hope it will be helpful

what does 1/10 + 1/100 equal?

Answers

in what terms? in decimal form, it would be .11

Answer:

0.11

Step-by-step explanation:

[tex] \frac{1}{10} + \frac{1}{100} \\ \\ = \frac{10}{100} + \frac{1}{100} \\ \\ = \frac{11}{100} \\ \\ = 0.11[/tex]

Can someone please help with this. I’m stuck.

Answers

Answer:

567 ghbvc. the high but

Step-by-step explanation:

solve accordingly to the picture

hope it helps

Using a two-tailed test with an alpha of 0.05, a research reports an obtained Z-score of -2. The researcher should reject the null
hypothesis.
True
False

Answers

Since the absolute value of the z-score is greater than the critical value, the statement is true.

When do we reject or do not reject the null hypothesis?

For a two-tailed test:

If the absolute value of the test statistic is less than the critical value, we do not reject the null hypothesis.If it is more, we reject.

The critical value for a two-tailed test with [tex]\alpha = 0.05[/tex] is of |z| = 1.96.

In this problem, we have that Z = -2, hence |Z| = 2, which is greater than the critical value, hence we reject the null hypothesis and the statement is True.

More can be learned about an hypothesis test at https://brainly.com/question/26454209

write the following numbers in words (1) 396 624 (2) 5 200 090 (3) 4 606 220​

Answers

Answer:

1. three hundred ninety six thousand twenty four

2. five million two hundred thousand ninety

3. four million six hundred six thousand two hundred twenty

Mrs. Bradford, a tailor, received 25 meters of fabric as a gift. She used 3 meters of fabric to make herself a long skirt. She plans to turn the rest of the fabric into smaller girls' dresses that she can sell. If she needs 110 centimeters of fabric for every dress, how many dresses can she make?

Answers

Answer:

20 dresses

Make sure to label

Step-by-step explanation:

there is 100 centimeters in 1 meter she has 2200 centimeters of fabric left and need 110 for each dress she can make 20 dresses

click on the point f(-3)

Answers

Answer:

The point at f(-3) is at (-3,1).

Step-by-step explanation:

To find the y-value of a function at a specific point by using the graph, keep in mind that the x-axis is the axis that lies horizontally on the graph.

So, look along the horizontal line through the center until you find where x = -3. Now find the corresponding y-value to that x-value by looking above or below that point until you find where it intersects the graph. When you find that point, look to see what number the point corresponds with on the y-axis. In this case, it corresponds to y = 1.

Remember that ordered pairs are formatted as (x,y).

Hope this helps.

fractions equivalent to 14/32

Answers

Answer:

7/16, 21/48, 28/64

Step-by-step explanation:

Hope this helps

Find the differential of
f
(
x
,
y
)
=
x
2
+
y
3
at the point
(
2
,
1
)
.

Answers

[tex]f(x,y)=x^2+y^3\implies \left.\cfrac{}{} 2x+3y^2 \right]_{\stackrel{\textit{\small x=2}}{\textit{\small y=1}}}\implies 2(2)+3(1)^2\implies 7[/tex]

A bag of tokens contains 9 red, 6 green, and 5​ blue tokens. What is the probability that a randomly selected token is red and green?

Answers

Answer:

27/200

Step-by-step explanation:

Knowing that:

A bag of tokens contains 9 red, 6 green, and 5​ blue tokens. What is the probability that a randomly selected token is red and green?

Solve:

and = multiply

Total = 9 red + 6 green + 5 blue = 9 + 6 + 5 = 20

Total = 20 token

Thus,

There are 9 red and 6 green..

Hence,

9 out of 20 is red

6 out of 20 is green

Equation

9/20 × 6/20

Multiply Across:

9 × 6 = 54

20 × 20 = 400

54/400

Simplify

27/200

Therefore, the probability that a randomly selected token is red and green is 27/200.

~Lenvy~

The probability that a randomly selected token is red and green is 27/200 in fraction form and 0.135 in decimal form.

What is probability?

Probability is defined as the possibility of an event being equal to the ratio of the number of favorable outcomes and the total number of outcomes.

Given that a bag of tokens contains 9 red, 6 green, and 5​ blue tokens.

Total = 9 red + 6 green + 5 blue = 9 + 6 + 5 = 20

Total token = 20

P(E₁) is 9 out of 20 are red.

P(E₂) is 6 out of 20 are green.

⇒ P(E₁) = 9/20

⇒ P(E₂) = 6/20

Required probability P(E) = P(E₁)×P(E₂)

Required probability P(E) = 9/20 × 6/20

Required probability P(E) = 54/400

Required probability P(E) = 27/200 = 0.135

Therefore, the probability that a randomly selected token is red and green is 27/200.

Learn more about probability here:

brainly.com/question/11234923

#SPJ2

75 POINTS IF U GET THIS CORRECT!!!!!!!!!!!!!!!!!!!!!!!!!!!!!!!!!!!!!!!!!!!!!!!!!!!!!!!!!!!!!!!!!!!!!!!!!!!!!!!!!!!!!!!! James has a collection of 20 miniature cars and trucks. Cars make up 40% of the collection. If James adds only cars to his collection, how many cars must he add to make the collection 75% cars?

Answers

Answer:

28 cars

Explanation:

cars + trucks = 20

Let "n" be the cars added in the collection

cars : 20 * 40% = 8 carstrucks : 20 - 8 = 12 trucks

solve:

[tex]\rightarrow \sf \dfrac{8+n}{20+n} =75\%[/tex]

[tex]\rightarrow \sf {8+n} =(20+n)75\%[/tex]

[tex]\rightarrow \sf {8+n} =15+0.75n[/tex]

[tex]\rightarrow \sf {8-15} =0.75n-n[/tex]

[tex]\rightarrow \sf -7 =0.25n[/tex]

[tex]\rightarrow \sf n = 28[/tex]

Thus, he needs to add 28 cars in the collection to make the collection 75% cars.

Answer:

28 cars

Step-by-step explanation:

Total number of cars and trucks = 20

Cars make up 40% of the collection

⇒ Number of cars = 40% of 20

                              = 0.4 x 20

                              = 8 cars

⇒ Number of trucks = total collection - number of cars

                                  = 20 - 8

                                  = 12 trucks

To calculate how many cars must be added to make the collection 75% cars, set up a ratio equation with x being the number of cars to be added:

(Added cars + existing cars) to total collection = 75%

⇒ x + 8 : x + 8 + 12 = 75 : 100

⇒ x + 8 : x + 20 = 75 : 100

[tex]\sf \implies \dfrac{x+8}{x+20}=\dfrac{75}{100}[/tex]

⇒ 100(x + 8) = 75(x + 20)

⇒ 100x + 800 = 75x + 1500

⇒ 100x -75x = 1500 - 800

⇒ 25x = 700

⇒ x = 28

Therefore he would need to add 28 cars to his collection to make his collection 75% cars.

Check

Number of cars = 8 + 28 = 36

Number of trucks = 12

Total collection = 36 + 12 = 48

Number of cars / total collection = 36/48 = 0.75 = 75%

Other Questions
Can someone help me? Q. 1 MWH is equal to ------- joulesa.3.6*10^10b.3.6*10^6c.3.6*10^9d.3.6 name a right angle I need some help How many patients had more than 5 cavities? I have my last 3 electrons on the third energy level. What are the coordinates? Rose is 60 inches tall. how many feet tall is Rose PLS HELP! 15 POINTS! Which of these would Congress look to do during a recession, as it pertains to fiscal policy?a. Raise taxesb. Raise government spendingc. Decrease the discount rated. Print more money He.... always....me. (have, support) A __________ is the amount of energy needed to raise the temperature of one __________ of water, ________ degree celcius.Write the correct words for those missing above (there are two possible correct answers).Your answer: Help meee pleaseee.i dont understand this exercisee What happens when red, blue, and green light come together? I hope you have a wonderful daywould you rather own a pet dragon or a UnicornI hope this question makes your day in any way(please don't delete my question) The World Health Organization (WHO) (1948) cited in Somerville et al. (2016: 35) defined health as a state of complete physical, mental and social well-being and not just the absence of illness or infirmity. - Identify 2 strengths and 2 limitations of the given definition what causes the outer layer of star to expand? 48 ounce bag of granola each month how many pounds if granola does she eat each year Tim read 70% of his book containing 230 pages. How many pages does he have left? What is the distance from the shore to the waterfall in the image of the koi pond shown below? All measurements are in yards. Round to the nearest hundredth if needed.3 yards99.72 yards13.8 yards6.43 yards In STU, the measure of U=90, SU = 36, UT = 77, and TS = 85. What ratio represents the secant of T?